Variation of Parameters/Wronskian

  • Thread starter Thread starter smashyash
  • Start date Start date
  • Tags Tags
    Variation
Click For Summary
The discussion revolves around solving the differential equation y'' + 3y' + 2y = 4e^(x) using the method of variation of parameters. The user correctly identifies the complementary solution Yc and computes y1 and y2. However, confusion arises regarding the use of the Wronskian in finding the particular solution, as the textbook answer suggests it is not needed. Clarification is provided that the Wronskian is indeed used to derive u1' and u2', which are then integrated to obtain the particular solutions. The user is encouraged to focus on integrating the expressions for u1' and u2' to complete the solution process.
smashyash
Messages
28
Reaction score
0
So I'm doing some practice problems to prepare for a test on Friday and I'm just curious about this problem::

y'' + 3y' + 2y = 4e^(x)

in factoring using characteristics:

(r+2)(r+1) = 0
r = -2,-1

so Yc = C1*e^(-2x) + C2*e^(x)
y1= e^(-2x)
y2= e^(-x)

(skipping some algebra)..I used these to find u1' and u2':

u1' = -4e^(3x)
u2' = 4e^(2x)

Now, I thought that this was the part where you use y1, y1', y2, y2' do find the wronskian and divide u1' and u2' by w(y1,y2) but apparently no because the answer given in the back of the book tells me that the wronskian isn't even used... why is that? I thought the wronskian was included in the equation::

Yp = -y1(x) * INT( (y2(x)*f(x) ) / W(x) ) dx + y2(x) INT( (y1(x)*f(x) ) / W(x) ) dx

Maybe I'm not understanding this equation as well as I thought I did.. Any comments?? Thanks!
 
Physics news on Phys.org
You use the Wronskian to find your u1' and u2':

u1' = - y2*f(x)/W
u2' = y1*f(x)/W

Then just integrate them to get the particular solutions.
 
Question: A clock's minute hand has length 4 and its hour hand has length 3. What is the distance between the tips at the moment when it is increasing most rapidly?(Putnam Exam Question) Answer: Making assumption that both the hands moves at constant angular velocities, the answer is ## \sqrt{7} .## But don't you think this assumption is somewhat doubtful and wrong?

Similar threads

  • · Replies 7 ·
Replies
7
Views
2K
  • · Replies 2 ·
Replies
2
Views
3K
Replies
2
Views
2K
  • · Replies 9 ·
Replies
9
Views
2K
  • · Replies 1 ·
Replies
1
Views
2K
  • · Replies 4 ·
Replies
4
Views
2K
  • · Replies 5 ·
Replies
5
Views
2K
Replies
1
Views
2K
  • · Replies 1 ·
Replies
1
Views
2K
  • · Replies 1 ·
Replies
1
Views
1K